Answer:
C. a 180° rotation about its center
Step-by-step explanation:
The question is on rotation of an object about the origin through 180°
When a point B, (h,k) is rotated about the origin 0 through 180° both clockwise and anticlockwise directions , the new position is B'(-h,-k)
Example, if we have point K (5,8) rotated through 180°, clockwise or anti-clockwise about the origin the new position will be; K'(-5,-8)
A 180° rotation about the center of a parallelogram will always map the parallelogram back onto itself due to the shape's symmetry.
Explanation:The transformation that will always map a parallelogram onto itself is a 180° rotation about its center. This is because a 180° rotation essentially flips the shape over, but, because a parallelogram is symmetrical, it would still look the same. For example, imagine a parallelogram ABCD with point A at the top and AB as the base. If you rotate it 180°, point A was at the top and now is at the bottom, but the shape is still a parallelogram. Similarly, the other transformations may not necessarily map the shape back onto itself.
Learn more about Parallelogram Transformation here:https://brainly.com/question/12456893
#SPJ3
Find the median of the following data set 6,2,59,12,11,9,9,54,54,46,2,32,43,11
By arranging the numbers in ascending order we get,
2, 2, 6, 9, 9, 11, 11, 12, 32, 43, 46, 54, 54, 59
The 2 middle numbers are:
11, 12
To find the median we have to add them and divide them by 2
So, (11+12)÷2
23÷2
11.5
For this case we have by definition that the median of a set of numbers is the average number in the set, after the numbers have been arranged from the lowest to the highest. If there is an even number of data, the median is the average of the two average numbers. So:
We order the numbers from least to greatest:
2,2,6,9,9,11,11,12,32,43,46,54,54,59
We have 14 numbers:
[tex]\frac {11 + 12} {2} = \frac {23} {2} = 11.5[/tex]
Answer:
11.5
The table gives the probability distribution of the number of DVDs sold per day at a music store. What is the probability of 5 or more DVDs being sold on any given day?
number of DVDs sold in a day
0-4
5-9
10-14
15-20
probability
0.1875
?
0.4375
0.125
A.
0.4375
B.
0.2525
C.
0.8125
D.
1.0000
Answer:
C. 0.8125
Step-by-step explanation:
The probability to sell 5 or more DVDs in a day is the complement of the probability of selling less than 5 DVDs in a day, since sales <5 DVDs + sales >= 5 DVDs = 100% of the sales.
So, we know what's the probability of selling less than 5 DVDs in a day is... it's 0.1875
So, to find the possibility of selling 5 or more DVDs in a day, we take 100% of the chances (1.0) and we subtract the probability of selling <5 DVDs in a day:
P = 1.0 - 0.1875 = 0.8125
C. 0.8125 is your answer.
I need help fast.
Number 18 and 19
Answer:
18.=400%
Step-by-step explanation:
80/20 we get 4 then we multiple 4 ×100 we get 400 and we check the answer 400% ×20 =80
A right cylinder has a radius of 4 and a height of 11 what’s the surface area
Answer:
[tex]A=376.99\ units^2[/tex]
Step-by-step explanation:
By definition, the surface area of a cylinder is given by the following formula:
[tex]A = 2\pi r h + 2\pi r ^ 2[/tex]
Where r is the radius and h is the height.
In this case, we have to:
[tex]r = 4\\\\h = 11[/tex]
then the surface area is:
[tex]A = 2\pi(4)(11) + 2\pi(4) ^ 2[/tex]
[tex]A=120\pi[/tex]
[tex]A=376.99\ units^2[/tex]
120pi units^2
Step-by-step explanation:
help me with this one
Answer:
I think that a pair could be the second choice. MTS and PWQ.
Step-by-step explanation:
Estimate the sum by first rounding each mixed number to the nearest whole number and then adding
3 8/9 + 2 5/6
For this case we have:
[tex]3 \frac {8} {9} = \frac {27 + 8} {9} = \frac {35} {9} = 3.8888[/tex]
Round the nearest whole, that is, to 4.
[tex]2 \frac {5} {6} = \frac {12 + 5} {6} = \frac {17} {6} = 2.8333[/tex]
Round the nearest whole, that is, to 3.
So, rewriting we have:
[tex]4 + 3 = 7[/tex]
ANswer:
[tex]3 \frac {8} {9} +2 \frac {5} {6} = 7[/tex]
twice a number subtracted from 35 is 9.what is the number
Ok, so equation for this would be 2x-35=9, 2x because its twice a number, x. 35 because it is taking off of 35. 9 because the grand total is 9.
2x-35=9 First add 35 to 9
2x=44 Then divide 44 by 9
x= approximately 4.9
Plz help with financial algebra !!
Answer: She will be there 8 years.
Step-by-step explanation:
THIS I S EASY I JUST DON'T KNOW IT PLZZ ANYONE I REALLY NEED THIS ASP I PROMISE BRAIBLIEST ANYONE???????PLZZ HELP I DON'T KNOW WHAT TO DO !!!!!!!15PTS!!! WIL GIVE BRAINLIETS!!!!Which function rule represents the data in the table?
x –3 –2 –1 0 1
y –17 –14 –11 –8 –5
y = –3x – 8
y = 1/3x – 8
y = 3x – 8
y = 1/3x + 8
Answer: The answer is y=3x-8
Step-by-step explanation: All you have to do is plug in the x values and see what you get for y. For example: when you plug in -3 for x. y=3(-3)-8 than you multiply the -3 by 3 and that give you y=-9-8 which equals y=-17 so that shows you the equation works.
plugging in -2 for x
y=3(-2)-8
y=-6-8
y=-14
I hope this helps!!!
Answer:
y = 3x -8
Step-by-step explanation:
You can try a point in the function rules and see if it works. The first (x, y) pair is suitable for finding the right answer:
y = -3(-3) -8 = -1 ≠ -17 . . . . eliminates the first choice
y = 1/3(-3) -8 = -9 ≠ -17 . . . . eliminates the second choice
y = 3(-3) -8 = -17 . . . . . the third choice is viable
y = 1/3(-3) +8 = 7 ≠ -17 . . . . eliminates the last choice
__
Only the third choice is a possible solution. Checking other points from the table verifies this as the correct one.
Help plzzzzzzzzzzzzzz
Answer:
y=8x-3
Step-by-step explanation:
Let f(x)=x2+14x+36 .
What is the vertex form of f(x)?
What is the minimum value of f(x)?
Enter your answers in the boxes.
Answer:
see explanation
Step-by-step explanation:
The vertex form of f(x) is
f(x) = (x - h)² + k
where (h, k) are the coordinates of the vertex
To obtain this form use the method of completing the square
add/subtract ( half the coefficient of the x- term )² to x² + 14x
f(x) = x² + 2(7)x + 49 - 49 + 36
= (x + 7)² - 13
The minimum value of f(x) is the y- coordinate of the vertex
vertex = (- 7, - 13), that is minimum value = - 13
The vertex form of the function f(x)=x²+14x+36 is (x + 7)² - 13, with the vertex at (-7, -13). The minimum value of f(x) is -13.
Explanation:To convert the function f(x) = x²+ 14x + 36 into vertex form, we need to complete the square. First, factor out the coefficient of the x² term if it is not 1 (in this case it is 1, so no factoring is necessary), and then group the x terms.
f(x) = (x²+ 14x) + 36
Add and subtract the square of half of the coefficient in front of x within the parenthesis to complete the square:
f(x) = (x² + 14x + 49) - 49 + 36
Simplify:
f(x) = (x + 7)² - 13
Now the function is in vertex form, which is y = a(x - h)² + k, where (h, k) is the vertex. Here, the vertex is at (-7, -13).
The minimum value of the function f(x) occurs at the vertex since the coefficient of the x² term is positive, indicating a parabolic shape opening upwards. Therefore, the minimum value of f(x) is -13.
Learn more about Vertex Form here:
https://brainly.com/question/29124549
#SPJ2
Really help please desperate in need help a sister out please just help
Answer:
Not safe
Step-by-step explanation:
Using the sine ratio in the right triangle
let the angle be x, then
sinx° = [tex]\frac{opposite}{hypotenuse}[/tex] = [tex]\frac{16.5}{17}[/tex]
x = [tex]sin^{-1}[/tex] ([tex]\frac{16.5}{17}[/tex] ) ≈ 76.1°
Since 76.1° > 70° then not safe at this height
i have an 86% in math, my final is worth 25% of my grade. where would i be if i failed it?
Answer:
Depends
Step-by-step explanation:
Depends on how many questions you missed. I'm going to assume that you mean you somehow missed every question. So, 86% - 25% would be 46%.
a rectangle is graphed on the coordinate grid. which represents the equation of a that is perpendicular to side r?
Answer:
Fourth answer choice: y = x + 3
Step-by-step explanation:
Note that sides a and s are perpendicular to side r. Both sides a and s have positive slopes, so we can immediately eliminate answer choices 1 and 2.
If we extend side s up and to the left, it intersects the y-axis at (0, 3). Thus, the y-intercept is +3 and the desired equation is:
y = x + 3
Answer:
The answer is D
Step-by-step explanation:
A line has a slope of 9 and includes the points (t, 7) and (-10, -2). What is the value of t?
T=__
[tex]\bf (\stackrel{x_1}{t}~,~\stackrel{y_1}{7})\qquad (\stackrel{x_2}{-10}~,~\stackrel{y_2}{-2}) \\\\\\ slope = m\implies \cfrac{\stackrel{rise}{ y_2- y_1}}{\stackrel{run}{ x_2- x_1}}\implies \cfrac{-2-7}{-10-t}=\stackrel{\stackrel{slope}{\downarrow }}{9}\implies \cfrac{-9}{-10-t}=9 \\\\\\ -9=-90-9t\implies 81=-9t\implies \cfrac{81}{-9}=t\implies -9=t[/tex]
T=-9 and the work is there
If you have any questions about what I did
Hope this helps.if it does please mark brainliest
Find (f-g)(x) when f(x) = 2x-3 and g(x) = -4x+8
Answer:
6x - 11
Step-by-step explanation:
(f - g)(x) = f(x) - g(x)
f(x) - g(x) = 2x - 3 - (- 4x + 8) = 2x - 3 + 4x - 8 = 6x - 11
Answer:
6x -11
Step-by-step explanation:
f(x) = 2x-3 and g(x) = -4x+8
f-g (x) =2x-3 - (-4x+8)
Distribute the minus sign
= 2x - 3 +4x -8
Combine like terms
= 6x -11
I Need Help can anyone figure out the answer to this problem a store sells cans of tomatoes price to show what size can of tomatoes has the lowest cost per ounce
Answer: 15 ounces
Step-by-step explanation:
We need to find the unit rate for each of the prices/ounces,
0.89/10 = 0.089
1.29/15 = 0.086
2.26/18 = 0.125...
3.39/32 = 0.1059375
The answer is:
B. 15 ounces
Because the unit rate(cost per ounce) is 0.086 cents per ounce which is less than:
A. 0.086
C. 0.126
D. 0.106
Please help! I'm am not that great at graphing like this...
Thanks a bunch! :)
Answer:
Step-by-step explanation:
If it's less than or greater than (<, >), then the circle is hollow.
If it's less than or equal to, or greater than or equal to (≤, ≥), then the circle is solid.
Less than points to the left, greater than points to the right.
Here we have less than (not equal to). So the circle is hollow, and the arrow points left. So the correct answer is the first one.
We can check our answer by picking a point and seeing if it's true. If we pick c=-6, is it true that -6 < -1? Yes.
I need help please and thank you
Answer:
Step-by-step explanation: if she runs 1 lap in 6 minutes, divide 66 by 6 to find how many laps she runs. this is 11. 11 is the minimum amount of laps that she runs.
n≥11
Answer:
11 ≤ n
Explanation:
[tex] \frac{66}{6} = 11[/tex]
More than indicates the minimum value, so you use the greater than or equal to.
The above answer is written in reverse, which the exact same result.
I am joyous to assist you anytime.
can two numbers have the same prime factorization
The prime factorization of a number is the set of prime numbers that can be multiplied together to give you that number. Therefore, two different numbers cannot have the same prime factorization, because if you multiply the same set of numbers together, you must get the same answer. For example, 2 × 3 × 5 will always give you 30.
Therefore, the answer will be no.
The point-slope form of the equation of a line that passes through points (8, 4) and (0, 2) is y – 4 = (x – 8). What is the slope-intercept form of the equation for this line?
y = x – 12
y = x – 4
y = x + 2
y = x + 6
Answer:
y = x - 4
Step-by-step explanation:
Rewrite y – 4 = (x – 8) in slope-intercept form. To do this, solve the given equation for y:
y - 4 = x - 8 → y = 4 + x - 8, or y = x - 4
As the point-slope form is given, we will simplify it;
[tex]y-4=x-8[/tex]
By adding +4 on both the sides, we get:
⇒[tex]y=x-8+4[/tex]
⇒[tex]y=x-4[/tex]
The slope-intercept form of the equation for this line is y=x-4.
Hence, the correct answer is option (b).
What is slope y-intercept form?The slope intercept formula y = mx + b is used when you know the slope of the line to be examined and the point given is also the y intercept (0, b). In the formula, b represents the y value of the y intercept point.
What is slope-intercept form example?Examples. y = 5x + 3 is an example of the Slope Intercept Form and represents the equation of a line with a slope of 5 and and a y-intercept of 3. y = −2x + 6 represents the equation of a line with a slope of −2 and and a y-intercept of 6.
Learn more about slope-intercept, refer to:
https://brainly.com/question/1884491
#SPJ2
What is the answer to x-9<15
Answer:
the answer is anything less than 24
Step-by-step explanation:
[tex]x - 9 < 15 = x - 9 + 9 < 15 + 9[/tex]
[tex] \times < 24[/tex]
One of the factors of the polynomial x^3 + 5x^2 + 6x is (x^2 + 3x). What is the other factor?
A. X-2
B. X + 2
C. X + 1
D. X - 1
Answer:
B
Step-by-step explanation:
HELPPP
The temperature inside a freezer is 2 degrees Celsius. What change can be made to the temperature inside the freezer so that it is 0 degrees Celsius?
A. increase the temperature by 2 degrees Celsius
B. decrease the temperature by 2 degrees Celsius
C. divide the temperature, in degrees Celsius, by 2
D. multiply the temperature, in degrees Celsius, by 2
Answer:
The answer would be B. decrease the temperature by 2 degrees Celsius.
Step-by-step explanation:
Since 2°C - 2°C= 0, then you would obviously have to subtract to find your answer.
Stuck, need help please.
Answer:
22.5 lbs
Step-by-step explanation:
First you find the cubic feet. That is the volume of the object which is length*width*height.
In your example, they are using inches. So you have to convert all units to feet. That would be 0.5 inches, 0.5 inches, and 5/3 inches. The volume would be 0.5*0.5*5/3 which is about 5/12 cubic feet
You then take that 5/12 cubic feet and multiply it by the 54 lbs/cubic foot. The answer would be 22.5 lbs
If x = -3 is the only x-intercept of the graph of a quadratic equation, which statement best describes the discriminant of the
equation?
The discriminant is negative
The discriminant is -3.
The discriminant is 0.
The discriminant is positive
Answer:
The discriminant is 0
Step-by-step explanation:
Since the graph of the quadratic equation has only one x-intercept, we can conclude that the quadratic has only one real root.
If a quadratic equation has only one real root, then the discriminant is 0
If x = -3 is the only x-intercept of the graph of a quadratic equation then the discriminant is 0
Is 3i a complex number
Remember that a complex number has the form a + bi. You need to figure out what a and b need to be. Since −3i is an imaginary number, it is the imaginary part (bi) of the complex number a + bi. This imaginary number has no real parts, so the value of a is 0.
ANSWER
Yes
EXPLANATION
The number
[tex]3i[/tex]
can be rewritten as
[tex]0 + 3i[/tex]This is of ty form
[tex]z = a + bi[/tex]
where 0 is the real part and 3i is the imaginary or complex part.
[tex]3i[/tex]
is therefore a complex number. It is purely imaginary because the real part is zero.
What is 2x -4 ≤ 4x answer
Answer:
x ≥ -2
Step-by-step explanation:
2x - 4 ≤ 4x
Subtract 2x from both sides:
-4 ≤ 2x
Divide by 2:
-2 ≤ x
So -2 ≤ x, or x ≥ -2.
You can check your answer by picking a number greater than -2 and see if it works. For example, if x = 0:
0 - 4 ≤ 0
-4 ≤ 0
A diver who has a mass of 68 kg climbs to a diving platform that is 7.5 m above the surface of a pool. How much gravitational potential energy does the diver have in relation to the pool’s surface? 510 J 1912 J 3825 J 4998 J
Answer:
option D
4998 J
Step-by-step explanation:
Given in the question that,
mass of the diver = 68 kg
distance of the diver from the surface of a pool = 7.5 m
The gravitational potential energy of an object is given by
GPE = m(g)(h)
here,
m is the mass of the diver
g is the acceleration due to gravity = 9.81m/s²
h is the height from the surface
Plug values in the formula to calculate GPE
GPE = 68(9.8)(7.5)
= 4998 J
Therefore, the gravitational potential energy of the diver with respect to the surface of the pool is 4998 J
Final answer:
The diver has 4986 Joules of gravitational potential energy in relation to the pool's surface, with the closest answer choice being 4998 J, assuming rounding differences. Hence, the answer is D.
Explanation:
To calculate the gravitational potential energy (GPE) of the diver in relation to the pool's surface, we can use the formula GPE = mgh, where m is the mass of the diver, g is the acceleration due to gravity (9.8 m/s2), and h is the height of the diving platform above the pool's surface.
Substituting the diver's mass (68 kg) and the height of the diving platform (7.5 m) into the equation, we get:
GPE = (68 kg) times (9.8 m/s2) times (7.5 m)
GPE = 4986 Joules
Therefore, the diver has 4986 Joules of gravitational potential energy in relation to the pool's surface. Since this value isn't exactly one of the choices, it's likely due to rounding differences, and the closest provided option is 4998 J, which we could assume is the intended answer.
In circle C, what is m?
I assume the 38º measurement refers to the measure of arc BE. Then
[tex]37^\circ=\dfrac{m\widehat{AJ}-38^\circ}2\implies m\widehat{AJ}=112^\circ[/tex]
and so
[tex]32^\circ=\dfrac{112^\circ-m\widehat{FH}}2\implies m\widehat{FH}=\boxed{48^\circ}[/tex]
(I think the theorem I used here is called "angle of intersecting secants")